Gönderen Konu: Genelleştirilmiş IMO 2001 #2 {çözüldü}  (Okunma sayısı 701 defa)

Çevrimdışı Hüseyin Yiğit EMEKÇİ

  • Geo-Maniac
  • ********
  • İleti: 768
  • Karma: +2/-0
Genelleştirilmiş IMO 2001 #2 {çözüldü}
« : Aralık 02, 2023, 04:25:23 ös »
Genelleştirme 1
$a,b,c,\lambda$ pozitif reeller ($\lambda \geq 8$) olmak üzere


$$\dfrac{a}{\sqrt{a^{2}+\lambda bc}}+\dfrac{b}{\sqrt{b^{2}+\lambda ca}}+\dfrac{c}{\sqrt{c^{2}+\lambda ab}}\geq 1-\dfrac{\left(\sqrt[3]{\lambda ^2}-4\right)\left(\sqrt[3]{\lambda^2}+2\right)^2}{6\sqrt[3]{\lambda}\left(\lambda +2\sqrt[3]{\lambda}\right)+8}$$


olduğunu gösteriniz.
« Son Düzenleme: Ocak 18, 2024, 12:25:20 öö Gönderen: Hüseyin Yiğit EMEKÇİ »
''Uzman, çok dar bir alanda yapılabilecek tüm hataları yapmış kişidir.''   ~Niels Bohr

Çevrimdışı Hüseyin Yiğit EMEKÇİ

  • Geo-Maniac
  • ********
  • İleti: 768
  • Karma: +2/-0
Ynt: Genelleştirilmiş IMO Shortlist 2001 #A.6
« Yanıtla #1 : Aralık 02, 2023, 04:26:30 ös »
$$\lambda =8$$
verildiğinde sağ taraf $1$ olur ve problem IMO 2001 #2'ya dönüşür.
« Son Düzenleme: Ocak 18, 2024, 12:16:13 öö Gönderen: Hüseyin Yiğit EMEKÇİ »
''Uzman, çok dar bir alanda yapılabilecek tüm hataları yapmış kişidir.''   ~Niels Bohr

Çevrimdışı Hüseyin Yiğit EMEKÇİ

  • Geo-Maniac
  • ********
  • İleti: 768
  • Karma: +2/-0
Ynt: Genelleştirilmiş IMO 2001 #2
« Yanıtla #2 : Aralık 11, 2023, 02:42:23 ös »
Homojeniteden dolayı $abc=1$ diyebiliriz.
$$\dfrac{a}{\sqrt{a^2+\lambda bc}}+\dfrac{b}{\sqrt{b^2+\lambda ca}}+\dfrac{c}{\sqrt{c^2+\lambda ab}} =\sum_{cyc}{\sqrt{\dfrac{a^3}{a^3+\lambda abc}}}$$
$$=\sum_{cyc}{\sqrt{\dfrac{a^3}{a^3+\lambda}}}$$
$a=\dfrac{\sqrt[3]{\lambda}}{x},b=\dfrac{\sqrt[3]{\lambda}}{y},c=\dfrac{\sqrt[3]{\lambda}}{z}$ dönüşümlerini yapalım ($abc=1\Rightarrow xyz=\lambda$)
$$LHS = \sum_{cyc}{\sqrt{\dfrac{\dfrac{\lambda}{x^3}}{\lambda \left(\dfrac{1}{x^3}+1\right)}}}=\sum_{cyc}{\dfrac{1}{\sqrt{x^3+1}}}$$
elde ederiz. Ayrıca aritmetik-geometrik ortalamadan da bildiğimiz üzere
$$\sqrt{x^3+1}=\sqrt{\left(x+1\right)\left(x^2-x+1\right)}\overbrace{\leq}^{AGO} \dfrac{\left(x+1\right)+\left(x^2-x+1\right)}{2}=\dfrac{x^2+2}{2}$$
Dolayısıyla sorumuz aslında $xyz=\lambda$ olmak üzere aşağıdaki ifadenin minimum değerini bulmak oluyor.
$$\sum_{cyc}{\dfrac{1}{\sqrt{x^3+1}}}\geq \dfrac{2}{x^2+2}+\dfrac{2}{y^2+2}+\dfrac{2}{z^2+2}\geq k$$
$k$ olsun. Soruda verilen $\lambda \geq 8$ ifadesi ile
$$k=1-\dfrac{\left(\sqrt[3]{\lambda ^2}-4\right)\left(\sqrt[3]{\lambda^2}+2\right)^2}{6\sqrt[3]{\lambda}\left(\lambda +2\sqrt[3]{\lambda}\right)+8}\leq 1$$
olduğunu söyleyebiliriz. Şimdi $k$ nın $1$'den küçük olmasını kullanabiliriz.
$$2\left(x^2y^2+y^2z^2+z^2x^2\right)+8\left(x^2+y^2+z^2\right)+24\geq 2k\left(x^2y^2+y^2z^2+z^2x^2\right)+4k\left(x^2+y^2+z^2\right)+k\left(x^2y^2z^2\right)+8k$$
$$\Rightarrow 2\left(1-k\right)\left(x^2y^2+y^2z^2+z^2x^2\right)+4\left(2-k\right)\left(x^2+y^2+z^2\right)+8\left(3-k\right)\geq x^2y^2z^2=\lambda^2$$

$k\leq 1$ olduğundan sol taraftaki ifadelerde aritmetik geometrik ortalama uygularsak

$$2\left(1-k\right)\left(x^2y^2+y^2z^2+z^2x^2\right)+4\left(2-k\right)\left(x^2+y^2+z^2\right)+8\left(3-k\right)\geq 2\left(1-k\right)\left(3\lambda\sqrt[3]{\lambda}\right)+4\left(2-k\right)\left(3\sqrt[3]{\lambda ^2}\right)+8\left(3-k\right)$$
$$=6\sqrt[3]{\lambda}\left(\lambda +4\sqrt[3]{\lambda}\right)-6k\sqrt[3]{\lambda}\left(\lambda+2\sqrt[3]{\lambda}\right)+24-8k\geq \lambda^2$$
$$k\left(6\sqrt[3]{\lambda}\left(\lambda +2\sqrt[3]{\lambda}\right)+8\right)\leq 6\sqrt[3]{\lambda}\left(\lambda+4\sqrt[3]{\lambda}\right)+24-\lambda^2$$
Dolayısıyla $k$ nın en iyi değeri yani problemin sol tarafının minimum değeri
$$k=\dfrac{6\sqrt[3]{\lambda}\left(\lambda+4\sqrt[3]{\lambda}\right)+24-\lambda^2}{\left(6\sqrt[3]{\lambda}\left(\lambda +2\sqrt[3]{\lambda}\right)+8\right)}=\dfrac{6\sqrt[3]{\lambda}\left(\lambda+2\sqrt[3]{\lambda}\right)+8+12\sqrt[3]{\lambda^2}+16-\lambda^2}{\left(6\sqrt[3]{\lambda}\left(\lambda +2\sqrt[3]{\lambda}\right)+8\right)}=1-\dfrac{\left(\sqrt[3]{\lambda ^2}-4\right)\left(\sqrt[3]{\lambda^2}+2\right)^2}{6\sqrt[3]{\lambda}\left(\lambda +2\sqrt[3]{\lambda}\right)+8}$$
elde edilir.
« Son Düzenleme: Ocak 18, 2024, 12:25:50 öö Gönderen: Hüseyin Yiğit EMEKÇİ »
''Uzman, çok dar bir alanda yapılabilecek tüm hataları yapmış kişidir.''   ~Niels Bohr

Çevrimdışı Hüseyin Yiğit EMEKÇİ

  • Geo-Maniac
  • ********
  • İleti: 768
  • Karma: +2/-0
Ynt: Genelleştirilmiş IMO 2001 #2 {çözüldü}
« Yanıtla #3 : Eylül 05, 2024, 02:51:38 öö »
Genelleştirme 2
Herhangi $(a)_1^n$ pozitif reelleri için

$$\sum_{cyc}{\dfrac{a_1}{\sqrt[n-1]{a_1^{n-1}+\left(n^{n-1}-1\right)a_2a_3\cdots a_n}}}\geq 1$$

eşitsizliği geçerlidir.
« Son Düzenleme: Eylül 05, 2024, 08:04:59 ös Gönderen: Hüseyin Yiğit EMEKÇİ »
''Uzman, çok dar bir alanda yapılabilecek tüm hataları yapmış kişidir.''   ~Niels Bohr

 


Sitemap 1 2 3 4 5 6 7 8 9 10 11 12 13 14 15 16 17 18 19 20 21 22 23 24 25 26 27 28 29 30 31 32 33 34 35 36 37 
SimplePortal 2.3.3 © 2008-2010, SimplePortal